Last visit was: 26 Apr 2024, 18:07 It is currently 26 Apr 2024, 18:07

Close
GMAT Club Daily Prep
Thank you for using the timer - this advanced tool can estimate your performance and suggest more practice questions. We have subscribed you to Daily Prep Questions via email.

Customized
for You

we will pick new questions that match your level based on your Timer History

Track
Your Progress

every week, we’ll send you an estimated GMAT score based on your performance

Practice
Pays

we will pick new questions that match your level based on your Timer History
Not interested in getting valuable practice questions and articles delivered to your email? No problem, unsubscribe here.
Close
Request Expert Reply
Confirm Cancel
SORT BY:
Kudos
Tags:
Show Tags
Hide Tags
Senior Manager
Senior Manager
Joined: 13 Mar 2021
Posts: 338
Own Kudos [?]: 101 [0]
Given Kudos: 227
Send PM
Intern
Intern
Joined: 29 Mar 2020
Posts: 40
Own Kudos [?]: 3 [0]
Given Kudos: 607
Location: India
GMAT 1: 720 Q48 V40
Send PM
Intern
Intern
Joined: 29 Mar 2020
Posts: 40
Own Kudos [?]: 3 [0]
Given Kudos: 607
Location: India
GMAT 1: 720 Q48 V40
Send PM
MBA House Admissions Consultant
Joined: 26 May 2022
Posts: 343
Own Kudos [?]: 70 [0]
Given Kudos: 0
Send PM
Re: Henry purchased 3 items during a sale. He received a 20 percent discou [#permalink]
Expert Reply
Henry purchased 3 items during a sale. He received a 20 percent discount off the regular price of the most expensive item and a 10 percent discount off the regular price of each of the other 2 items. Was the total amount of the 3 discounts greater than 15 percent of the sum of the regular prices of the 3 items?


(1) The regular price of the most expensive item was $50, and the regular price of the next most expensive item was $20

(2) The regular price of the least expensive item was $15
Attachments

1FC95E65-B977-4642-A180-C4B972CDEA48.jpeg
1FC95E65-B977-4642-A180-C4B972CDEA48.jpeg [ 1.13 MiB | Viewed 1429 times ]

Intern
Intern
Joined: 30 Jun 2022
Posts: 1
Own Kudos [?]: 0 [0]
Given Kudos: 1
Send PM
Re: Henry purchased 3 items during a sale. He received a 20 percent discou [#permalink]
DrVanNostrand wrote:
Bunuel wrote:
docabuzar wrote:
Henry purchased 3 items during a sale. He received a 20 percent discount off the regular price of the most expensive item and a 10 percent discount off the regular price of each of the other 2 items. Was the total discount of these three items greater than 15 percent of the sum of the regular prices of the 3 items?

(1) The regular price of the most expensive item was $50, and the regular price of the next most expensive item was $20
(2) The regular price of the least expensive item was $15


Let the regular prices be a, b, and c, so that a > b > c.

Basically the questions: is \(0.2a+0.1b+0.1c>0.15(a+b+c)\)? --> is \(a>b+c\)?

(1) The regular price of the most expensive item was $50 and the regular price of the next most expensive item was $20 --> \(a=50\), \(b=20\), \(c\leq{20}\) (as the second most expensive item was $20 then the least expansive item, the third one, must be less than or equal to 20). So the question becomes: is \(50>20+c\) --> is \(c<30\)? As we got that \(c\leq{20}\), hence the above is always true. Sufficient.

(2) The regular price of the least expensive item was $15. Clearly insufficient.

Answer: A.



Hi Bunuel. First of all, thanks for the tremendous help that you have been providing to us all.

Coming back to the question, I don't think the answer is A ( it should be C ). Algebraically, what you have said is right. However, if we take specific numerical values, the answer ranges from "Yes" ( for c= 20 ) to "No" ( for c = 10 ). Also, inequality provides us with a range of values. For some, the answer is ">15%" and for some that sanwer is "<15%".

Kindly explain.


Answer is A. Imagine A=50 B=20 C=20, when you calculate the total discount of the sum you do 90*0.15%= 13.5 . the discount of ABC is 14. so 14>13.5 . What happens if you reduce C? 14 goes down, but 13.5 goes down as well, as it is not 90*0.15% anymore, you'll have to subtract from 90 the same amount you subtract from C. Therefore A.
Senior Manager
Senior Manager
Joined: 16 Nov 2021
Posts: 476
Own Kudos [?]: 27 [0]
Given Kudos: 5900
Location: United Kingdom
Send PM
Re: Henry purchased 3 items during a sale. He received a 20 percent discou [#permalink]
docabuzar wrote:
Henry purchased 3 items during a sale. He received a 20 percent discount off the regular price of the most expensive item and a 10 percent discount off the regular price of each of the other 2 items. Was the total amount of the 3 discounts greater than 15 percent of the sum of the regular prices of the 3 items?


(1) The regular price of the most expensive item was $50, and the regular price of the next most expensive item was $20

(2) The regular price of the least expensive item was $15


Attachment:
Henry discount on items.GIF


Hi KarishmaB
Could you kindly explain this question with Weight Average or Allegation method please?
Thanks
Tutor
Joined: 16 Oct 2010
Posts: 14831
Own Kudos [?]: 64941 [0]
Given Kudos: 427
Location: Pune, India
Send PM
Re: Henry purchased 3 items during a sale. He received a 20 percent discou [#permalink]
Expert Reply
Kimberly77 wrote:
docabuzar wrote:
Henry purchased 3 items during a sale. He received a 20 percent discount off the regular price of the most expensive item and a 10 percent discount off the regular price of each of the other 2 items. Was the total amount of the 3 discounts greater than 15 percent of the sum of the regular prices of the 3 items?


(1) The regular price of the most expensive item was $50, and the regular price of the next most expensive item was $20

(2) The regular price of the least expensive item was $15


Attachment:
Henry discount on items.GIF


Hi KarishmaB
Could you kindly explain this question with Weight Average or Allegation method please?
Thanks


This is a 0 calculation 30 sec question in case you use weighted averages.

20% discount on the most expensive item
10% discount on the two cheaper ones

Discounts are given on regular prices (marked prices) of items so weights will be the regular prices.
When will the overall discount be 15%? When the weights of the two are equal (right in the centre of 20 and 10). This means that if the price of the most expensive item is equal to the price of the other two items combined, then overall discount will be 15%.

(1) The regular price of the most expensive item was $50, and the regular price of the next most expensive item was $20

This tells us that the price of the most expensive was $50 but the price of the next most expensive was $20 so the price of the cheapest item will be less than $20. Hence price of the two cheaper items will certainly be less than $50. Hence the average discount will be closer to 20% than to 10% i.e. the overall discount will be more than 15%. We can answer 'Yes'.
This statement alone is sufficient.

(2) The regular price of the least expensive item was $15

We don't know the prices of the other two items. Hence we cannot say what the overall discount will be.
This statement alone is not sufficient.

Answer (A)

Check this video for more on weighted averages and how to use them: https://youtu.be/_GOAU7moZ2Q
Senior Manager
Senior Manager
Joined: 16 Nov 2021
Posts: 476
Own Kudos [?]: 27 [0]
Given Kudos: 5900
Location: United Kingdom
Send PM
Re: Henry purchased 3 items during a sale. He received a 20 percent discou [#permalink]
KarishmaB wrote:
Kimberly77 wrote:
docabuzar wrote:
Henry purchased 3 items during a sale. He received a 20 percent discount off the regular price of the most expensive item and a 10 percent discount off the regular price of each of the other 2 items. Was the total amount of the 3 discounts greater than 15 percent of the sum of the regular prices of the 3 items?


(1) The regular price of the most expensive item was $50, and the regular price of the next most expensive item was $20

(2) The regular price of the least expensive item was $15


Attachment:
Henry discount on items.GIF


Hi KarishmaB
Could you kindly explain this question with Weight Average or Allegation method please?
Thanks


This is a 0 calculation 30 sec question in case you use weighted averages.

20% discount on the most expensive item
10% discount on the two cheaper ones

Discounts are given on regular prices (marked prices) of items so weights will be the regular prices.
When will the overall discount be 15%? When the weights of the two are equal (right in the centre of 20 and 10). This means that if the price of the most expensive item is equal to the price of the other two items combined, then overall discount will be 15%.

(1) The regular price of the most expensive item was $50, and the regular price of the next most expensive item was $20

This tells us that the price of the most expensive was $50 but the price of the next most expensive was $20 so the price of the cheapest item will be less than $20. Hence price of the two cheaper items will certainly be less than $50. Hence the average discount will be closer to 20% than to 10% i.e. the overall discount will be more than 15%. We can answer 'Yes'.
This statement alone is sufficient.

(2) The regular price of the least expensive item was $15

We don't know the prices of the other two items. Hence we cannot say what the overall discount will be.
This statement alone is not sufficient.

Answer (A)

Check this video for more on weighted averages and how to use them: https://youtu.be/_GOAU7moZ2Q


Brilliant explanation always KarishmaB thanks.
One question regarding 10% discount on the two cheaper ones, not sure why are we only using 10 for thinking here in below sentence and not 20 (as it's 10% each for 2 items) or other numbers ? Could you help clarify? Thanks

When will the overall discount be 15%? When the weights of the two are equal (right in the centre of 20 and 10)
Senior Manager
Senior Manager
Joined: 16 Nov 2021
Posts: 476
Own Kudos [?]: 27 [0]
Given Kudos: 5900
Location: United Kingdom
Send PM
Re: Henry purchased 3 items during a sale. He received a 20 percent discou [#permalink]
KarishmaB wrote:
Kimberly77 wrote:
Brilliant explanation always KarishmaB thanks.
One question regarding 10% discount on the two cheaper ones, not sure why are we only using 10 for thinking here in below sentence and not 20 (as it's 10% each for 2 items) or other numbers ? Could you help clarify? Thanks

When will the overall discount be 15%? When the weights of the two are equal (right in the centre of 20 and 10)


It comes down to knowing what the weights are in weighted averages. When we are averaging discount percentages, the weights will be marked price.

Discount percentage = Discount/Marked price

You should check this post first if you haven't seen it: https://anaprep.com/arithmetic-weights- ... d-average/

So it doesn't matter whether we have 1 item or 2 items or 4. What matters is the overall marked price on which we are giving the discount.

So if there are two discounts 20% and 10% and they average out to 15%, it means that the marked price on which 20% was given was equal to marked price on which 10% was given say $100 each. So 20% on 100 is $20 and 10% on 100 is $10 and overall we have $30 on $200 which is a discount of 15%.

It does not matter how many items there were. Say there was 1 item with marked price $100 on which 20% was given and there were 2 items with marked price $50 each on which 10% was given. Still discount would be 20 + 5 + 5 = $30 on marked price of $200 giving a 15% overall discount.
Even if we say we had 4 of the cheaper items of $25 each, still the logic stays the same.


Brilliant thanks KarishmaB and get it.
What an innovative way of your solutions always !!!
GMAT Club Bot
Re: Henry purchased 3 items during a sale. He received a 20 percent discou [#permalink]
   1   2 
Moderator:
Math Expert
92948 posts

Powered by phpBB © phpBB Group | Emoji artwork provided by EmojiOne